Multiple choice answers (1 Viewer)

00iCon

Member
Joined
Feb 1, 2009
Messages
383
Location
ISS
Gender
Male
HSC
2009
thats where i went wrong
i thought the force was down because that where its moving =S ahwell
NONNONONONONONONONONOMNONONONONONOJNONONONO!!!!
Force is the force that pushes it through the field, not the induced force!!!!
That being said, the same rule applies to a current carrying conductor in a B field...
 

Ostentatious

jfK1c45g2f4Of56Eyd4f57cxZ
Joined
Jan 23, 2009
Messages
324
Gender
Male
HSC
2009
NONNONONONONONONONONOMNONONONONONOJNONONONO!!!!
Force is the force that pushes it through the field, not the induced force!!!!
That being said, the same rule applies to a current carrying conductor in a B field...

What is the Force then? This was one of my stumpers.
 

tanuwidge

New Member
Joined
Mar 13, 2008
Messages
22
Gender
Male
HSC
2009
Nope, carefully look at the direction of the current between A and B.

Draw the direction.

Now draw the direction of the magnetic field lines (N to S)

You'll see they're always perpendicular.
Force changes with time so it can't be b.....
 

kieranjackson91

New Member
Joined
Mar 24, 2008
Messages
27
Gender
Male
HSC
2009
Did I really get 15/15?
awesome!
sorry to pop your bubble but it's pretty much decided that 9. is A. and you said 9 is C? If you think about it using lenz's law, you said current goes to P right? but that means the field created by this new current will cancel out with the current field BELOW the conductor, thus increasing the speed of the conductor down the page which goes against the law of conservation of energy.
11 is D

sin0=0 so force is minimum
as theta increases, force increases it is not constant till 90 degrees
sin90=1 so force is maximum

cos0=1 so initially torque is maximum
cos90=0 so torque is minimum

so its D

QED
read previous posts- angle is always 90 since its the angle between the conductor AB and the field lines from N to S which is always 90. It is force on AB, reason force is a straight line is because angle is always 90. torque however, approaches 0 as AB reaches the top. torques is the turning force around the axis so is always positive since the motor is always turning in one direction. so not QED. its qeB lol
 

tanuwidge

New Member
Joined
Mar 13, 2008
Messages
22
Gender
Male
HSC
2009
Nope, carefully look at the direction of the current between A and B.

Draw the direction.

Now draw the direction of the magnetic field lines (N to S)

You'll see they're always perpendicular.
Force changes with time so it can't be b.....and it's not based on the current. It's based on the coil itself which is initially parallel to the field so theta=0 . Therefore initial force is 0
 

00iCon

Member
Joined
Feb 1, 2009
Messages
383
Location
ISS
Gender
Male
HSC
2009
9 cannot be C

use your left hand because your dealing with electrons
I just think: electron flow is opposite to conventional current... every time. But the force's direction is the problem i have... maybe 14/15?
 

williamcarr1991

New Member
Joined
May 5, 2008
Messages
12
Gender
Male
HSC
2009
ok, i got caddcacccdddaca
was q3 c or d?
they still mean the same thing to me :p
 
Last edited:

Ostentatious

jfK1c45g2f4Of56Eyd4f57cxZ
Joined
Jan 23, 2009
Messages
324
Gender
Male
HSC
2009
Force changes with time so it can't be b.....and it's not based on the current. It's based on the coil itself which is initially parallel to the field so theta=0 . Therefore initial force is 0
But if you look carefully at the question it says force F on WIRE AB, so that's how you have to go about it.
 

kieranjackson91

New Member
Joined
Mar 24, 2008
Messages
27
Gender
Male
HSC
2009
Force changes with time so it can't be b.....and it's not based on the current. It's based on the coil itself which is initially parallel to the field so theta=0 . Therefore initial force is 0
question clearly says 'force F on wire AB' not coil. torque however is 'the torque T on the rotor'
 

Macdwg

Professional Mad Lad
Joined
Jun 15, 2008
Messages
93
Gender
Male
HSC
2009
9 cannot be C

use your left hand because your dealing with electrons
Actually i have a feeling that it is C.

If you use the right hand palm rule and point your palm in the direction of the velocity which is down the page, then point your fingers in as the mag field is going into the page, then the thumb gives the direction of the flow of HOLES known as conventional current. Thus the electrons move in the opposite direction and congregate at R.

Although debate it! I might be wrong because it could be eddy currents?
 

GT03

New Member
Joined
Nov 11, 2008
Messages
20
Gender
Male
HSC
2009
Actually i have a feeling that it is C.

If you use the right hand palm rule and point your palm in the direction of the velocity which is down the page, then point your fingers in as the mag field is going into the page, then the thumb gives the direction of the flow of HOLES known as conventional current. Thus the electrons move in the opposite direction and congregate at R.

Although debate it! I might be wrong because it could be eddy currents?
i think its eddy currents. because the conductor moves in the magnetic field, it experiences changing flux. so the current generated produces a force that OPPOSES the motion by lenz's and hence ur palm points UP. thus electrons are at P by right hand rule. anyone agree?
 

JAM91

New Member
Joined
Oct 23, 2009
Messages
29
Gender
Male
HSC
2009
wow i made some stupid fucking mistakes

also q2 according to all text books man has made centripetal force is 9.8 m/s towards the earth

question 4 is D due to the vx component (how the hell else would it have gone further)

question 5 i have no idea i naturally assumed since the moon orbits around the earth the sun would still have no impact at all considering the moon is in line with the sun every day and it plays no effect on it to my understanding....

9 i thought was C because according to the right hand principal the current would be induced towards the P side (THE CURRENT IS POSITIVE) thus the electrons would all head over to the R side

15 is B because an electric field goes from + to - and thus the work done to put the particle in position P is the greatest

feel free to argue :D
 

6tgyuio

New Member
Joined
Sep 21, 2009
Messages
6
Gender
Male
HSC
2010
can i ask the reason for question 4? I spent half an hour checking the maths with it and im not sure if its a or d. if v=u+at, for vertical u=0, meaning t=v/a. since d said arrive at same time but different velocities, could it be not true since at the same instant time velocities would have to be the same? that said for horizontal x=ut, so i guessed smaller time meant less distance...
 

Users Who Are Viewing This Thread (Users: 0, Guests: 1)

Top